12
$\begingroup$

Denote by $W$ the space of all symmetric bilinear forms on $\mathbb{R}^n$.

Let $Q$ be a quadratic form on $W$. Suppose that $Q(b)\geqslant 0$ for any $b\in W$ such that $b(X,Y)=\ell(X)\cdot\ell(Y)$ for some linear function $\ell$.

Is it possible to find a nonnegative quadratic form $\tilde Q$ such that $\tilde Q(b)=Q(b)$ for any $b$ as above?

Comments

  • The forms $b$ as above (up to sign) can be described by quadratic equations $$b(X,Y)\cdot b(Z,W)=b(X,Z)\cdot b(Y,W).$$ This leads to a more general version of this question, but it has a negative answer (thanks to Terry Tao).

  • I know that the answer is yes for $n=2$.

$\endgroup$

1 Answer 1

20
$\begingroup$

(6 October 2023) I'll leave the original argument below because it seems that many people liked it, but, in fact, it wanders around and introduces a lot of unnecessary information, which obscures the essential point, so here is a cleaned up version:

The answer 'yes' when $n\le 3$ but 'no' when $n\ge4$: Let $V = \mathbb{R}^n$, so $W = S^2(V^*)$ and the quadratic forms on $W$ are $S^2(W^*) = S^2\bigl(S^2(V^*)^*\bigr) = S^2(S^2(V))$ (since $S^2(V^*)$ is canonically isomorphic to $S^2(V)^*$). The (linear) 'multiplication map' $\mu:S^2(S^2(V))\to S^4(V)$ is surjective. Thus, each quadratic form on $W$ defines a quartic function on $V^*$, and every quartic function on $V^*$ is of the form $\mu(Q)$ for some $Q\in S^2(W^*)$.

The question is whether every non-negative quartic function on $V^*$ is of the form $\mu(Q)$ for some non-negative quadratic form $Q \in S^2(W^*)$. This is equivalent to asking whether every non-negative quartic function on $V^*$ is a sum of squares of quadratic functions on $V^*$.

However, it is known that for $n\ge4$, there exist non-negative quartic polynomials $P$ on $V^*$ that cannot be written as a sum of squares of quadratic polynomials on $V^*$, while, for $n\le 3$, any non-negative quartic polynomial $P$ on $V^*$ can be written as a sum of squares of quadratic polynomials.

Original Argument:

The answer 'yes' when $n\le 3$ but 'no' when $n\ge4$. Here is why:

Let $V = \mathbb{R}^n$, so $W = S^2(V^*)$ and the quadratic forms on $W$ are $S^2(W^*) = S^2\bigl(S^2(V^*)^*\bigr)$. It is known that there is a canonical $\mathrm{GL}(V)$-invariant exact sequence $$ 0\longrightarrow \Lambda^4(V)\longrightarrow S^2\bigl(\Lambda^2(V)\bigr)\longrightarrow S^2\bigl(S^2(V^*)^*\bigr)\longrightarrow S^4(V)\longrightarrow 0, $$ thus, there is a $\mathrm{GL}(V)$-module $K(V)$ such that $$ S^2\bigl(\Lambda^2(V)\bigr) = \Lambda^4(V)\oplus K(V) \qquad\text{and}\qquad S^2\bigl(S^2(V^*)^*\bigr) = S^4(V)\oplus K(V). $$ The module $K(V)$ is $\mathrm{GL}(V)$-irreducible and of dimension $n^2(n^2{-}1)/12$. It is exactly the set of quadratic forms $Q$ on $W$ that vanish on all of the symmetric bilinear forms on $V$ of the form $B(v,w) = \ell(v)\ell(w)$ for some $\ell\in V^*$. (The quadratic forms with this latter property must be some $\mathrm{GL}(V)$-invariant submodule of $S^2\bigl(S^2(V^*)^*\bigr)$, and it clearly does not contain $S^4(V)$.)

Suppose that $Q$ be nonnegative on all of the rank 1 elements of $W$. This is equivalent to the condition that, when we write $Q = Q' + Q''$, with $Q'\in S^4(V)$ and $Q''\in K(V)$, then $Q'$, when considered as a quartic polynomial on $V^*$, be a nonnegative quartic polynomial.

If $\tilde Q$ were a non-negative quadratic form on $W$, then it would be a sum of squares of linear functions on $W$, and hence, if $\tilde Q - Q$ lay in $K(V)$, it would follow that $Q'$ would be a sum of squares of quadratic functions on $V^*$.

However, it is known that for $n\ge4$, there exist non-negative quartic polynomials $Q'$ on $V^*$ that cannot be written as a sum of squares of quadratic polynomials on $V^*$.

Thus, for $n\ge 4$, the answer to the OP's question is 'no'.

Meanwhile, when $n\le 3$, every non-negative quartic polynomial in $n$ variables is a sum of squares of quadratic polynomials in those $n$ variables, so the above analysis shows that, when $Q = Q' + Q''$ where $Q'\in S^4(V)$ is a nonnegative function on $V^*$ and $Q''\in K(V)$, there does exist a nonnegative quadratic $\tilde Q\in S^2\bigl(S^2(V^*)^*\bigr)$ such that $\tilde Q - Q$ lies in $K(V)$.

$\endgroup$
7
  • 3
    $\begingroup$ Thank you very much. I guess you wanted to say, but did not say that $K(V)$ is the space of curvature tensors on $V$. $\endgroup$ Oct 1 at 0:02
  • 6
    $\begingroup$ @AntonPetrunin: Yes, of course that's true. It's interesting that $K(V)$ is irreducible as a $\mathrm{GL}(V)$-module, whereas it's reducible as an $\mathrm{SO}(V)$-module (which is the way we usually think of it). $\endgroup$ Oct 1 at 0:24
  • 2
    $\begingroup$ Also, for another way that this $\mathrm{GL}(V)$ modules shows up, see my answer at mathoverflow.net/q/100372. $\endgroup$ Oct 1 at 0:36
  • 1
    $\begingroup$ @PeterLeFanuLumsdaine: There's not that much to say. The usual multiplicity formulae in $\mathrm{GL}(V)$-representation theory show that $S^2(\Lambda^2(V))\simeq \Lambda^4(V)\oplus K(V)$ while $S^2(S^2(V))\simeq S^4(V)\oplus K(V)$, where $K(V)$ is irreducible. The center map of the exact sequence on decomposable elements is $$(v_1\wedge v_2)\circ(v_3\wedge v_4)\longmapsto (v_1\circ v_3)\circ(v_2\circ v_4)- (v_1\circ v_4)\circ(v_2\circ v_3).$$ It's clearly nonzero, so it has to match the two copies of $K(V)$, making $\Lambda^4(V)$ be the kernel and $S^4(V)$ be the cokernel. $\endgroup$ Oct 2 at 14:06
  • 1
    $\begingroup$ @PeterLeFanuLumsdaine: Oh, and I should have pointed out that, since $S^2(V^*)$ is canonically isomorphic to $\bigl(S^2(V)\bigr)^*$, we also have $S^2\bigl(S^2(V^*)^*\bigr)$ is canonically isomorphic to $S^2(S^2(V))$. $\endgroup$ Oct 2 at 14:10

Your Answer

By clicking “Post Your Answer”, you agree to our terms of service and acknowledge that you have read and understand our privacy policy and code of conduct.

Not the answer you're looking for? Browse other questions tagged or ask your own question.